Получение перекрестного произведения из алгебры углового момента

Можно ли вывести:

л ^ "=" р ^ × п ^
из алгебры углового момента:
[ л ^ я , л ^ Дж ] "=" я     ϵ я Дж к л ^ к   ?
Я где-то читал, что коммутационные соотношения вида
[ а я , б Дж ] "=" ϵ я Дж к с к
допускают «естественное переписывание в терминах кросс-произведений», но никаких подробностей по поводу этого утверждения не было.

Ответы (3)

Невозможно получить орбитальный угловой момент л "=" р × п из с о ( 3 ) только коммутационными соотношениями, поскольку спиновый оператор С также выполняет те же коммутационные соотношения, но, конечно, отличается от р × п .

Это именно то, что я бы написал, но я не знаю, нужно ли говорить что-то еще, чтобы объяснить, почему или как мы знаем, что это вращение отличается.

Я где-то читал, что коммутационные соотношения вида

[ а я , б Дж ] "=" ϵ я Дж к с к
допускают «естественное переписывание в терминах кросс-произведений», но никаких подробностей по поводу этого утверждения не было.

Это «естественное переписывание» канонических коммутационных соотношений для угловых моментов в терминах перекрестных произведений:

Дж × Дж "=" я Дж

То есть, «естественное переписывание» относится к переписыванию самих коммутационных соотношений в терминах перекрестного произведения, а не переписывании отдельного углового момента в терминах перекрестных произведений других величин.

Явно, для углового момента:

Дж × Дж "=" я Дж
Таким образом:
ϵ я Дж к Дж Дж Дж к "=" я Дж я
Таким образом:
ϵ я л м ϵ я Дж к Дж Дж Дж к "=" я ϵ я л м Дж я
Таким образом:
Дж л Дж м Дж м Дж л "=" я ϵ я л м Дж я
То есть:
[ Дж л , Дж м ] "=" я ϵ л м я Дж я

С точки зрения ваших «a», «b» и «c» аналогичное «естественное переписывание»:

а × б + б × а "=" 2 с

Вы можете увидеть это, рассмотрев:

( а × б ) я "=" ϵ я Дж к а Дж б к "=" ϵ я Дж к ( б к а Дж + [ а Дж , б к ] )
"=" ϵ я к Дж б к а Дж + ϵ я Дж к [ а Дж , б к ]
"=" ( б × а ) я + ϵ я Дж к ϵ Дж к л с л
"=" ( б × а ) я + 2 дельта я л с л
"=" ( б × а + 2 с ) я

Как [ а л , б м ] "=" а л б м а м б л ?
упс, хороший вопрос... как-то облажался... Позвольте мне это исправить.

[ А ^ я , Б ^ Дж ] "=" ϵ я Дж к С ^ к , А ^ я Б ^ Дж Б ^ Дж А ^ я "=" ϵ я Дж к С ^ к , ϵ я Дж н ( А ^ я Б ^ Дж Б ^ Дж А ^ я ) "=" ϵ я Дж н ϵ я Дж к С ^ к , ϵ я Дж н А ^ я Б ^ Дж ϵ я Дж н Б ^ Дж А ^ я "=" 2 С ^ н , ( А ^ × Б ^ ) н + ( Б ^ × А ^ ) н "=" 2 С ^ н , 1 2 ( А ^ × Б ^ + Б ^ × А ^ ) "=" С ^ .
благодаря тому факту, что ϵ я Дж н ϵ я Дж к "=" 2 дельта н к .

Что не так с этим выводом?
Помимо того факта, что иногда вы явно пишете суммирование, а иногда — неявно, в выводе нет ничего плохого. Вероятно, люди голосуют против, потому что нет пояснительного текста к уравнениям... или, может быть, явное/неявное сбивает с толку...